2004 AMC 12A Problems/Problem 21

Revision as of 22:23, 3 December 2007 by Azjps (talk | contribs) (solution, that was surprisingly easy ..)
(diff) ← Older revision | Latest revision (diff) | Newer revision → (diff)

Problem

If $\sum_{n = 0}^{\infty}{\cos^{2n}\theta = 5$ (Error compiling LaTeX. Unknown error_msg), what is the value of $\cos{2\theta}$?

$\text {(A)} \frac15 \qquad \text {(B)} \frac25 \qquad \text {(C)} \frac {\sqrt5}{5}\qquad \text {(D)} \frac35 \qquad \text {(E)}\frac45$

Solution

This is an infinite geometric series, which sums to $\frac{\cos^0 \theta}{1 - \cos^2 \theta} = 5 \Longrightarrow 1 = 5 - 5\cos^2 \theta \Longrightarrow \cos^2 \theta = \frac{4}{5}$. Using the formula $\cos 2\theta = 2\cos^2 \theta - 1 = 2\left(\frac 45\right) - 1 = \frac 35 \Rightarrow \mathrm{(D)}$.

See also

2004 AMC 12A (ProblemsAnswer KeyResources)
Preceded by
Problem 20
Followed by
Problem 22
1 2 3 4 5 6 7 8 9 10 11 12 13 14 15 16 17 18 19 20 21 22 23 24 25
All AMC 12 Problems and Solutions